Формула включения-исключения — различия между версиями

Материал из Викиконспекты
Перейти к: навигация, поиск
(Формула включения-исключения)
(Теорема)
Строка 46: Строка 46:
 
<center>
 
<center>
 
<math> | A | = | A_n| + \Bigg( \sum_{I \subset \{ 1,2, \ldots ,n-1 \} }  (-1)^{|I|+1}  \Big| \bigcap_{ j \in I } A_j \Big| \Bigg) - \Bigg( \sum_{I \subset \{ 1,2, \ldots ,n-1 \} } (-1)^{|I|+1}  \Big| \bigcap_{ j\in I \cup \{ n \} } A_j \Big| \Bigg) = \sum_{I \subset \{ 1,2, \ldots ,n \} } (-1)^{k+1}  \Big| \bigcap_{ j \in I } A_j \Big| </math>
 
<math> | A | = | A_n| + \Bigg( \sum_{I \subset \{ 1,2, \ldots ,n-1 \} }  (-1)^{|I|+1}  \Big| \bigcap_{ j \in I } A_j \Big| \Bigg) - \Bigg( \sum_{I \subset \{ 1,2, \ldots ,n-1 \} } (-1)^{|I|+1}  \Big| \bigcap_{ j\in I \cup \{ n \} } A_j \Big| \Bigg) = \sum_{I \subset \{ 1,2, \ldots ,n \} } (-1)^{k+1}  \Big| \bigcap_{ j \in I } A_j \Big| </math>
</center>
 
 
== Теорема ==
 
Пусть <math> A = \bigcup_{i=1}^{n}A_i </math> , тогда по формуле включения-исключения:
 
<center>
 
<math> | A | = \sum_{I=(i_1,i_2, \ldots ,i_k) \subset \{ 1,2, \ldots ,n \} } (-1)^{k+1}  \Big| \bigcap_{ j \in I } A_j \Big| </math>
 
 
</center>
 
</center>
  

Версия 02:14, 8 ноября 2010

Формула включения-исключения

Формула включения-исключения - это комбинаторная формула, которая позволяет определить мощность объединения конечных множеств, если известны их мощности и мощности всех их возможных пересечений.

случай для двух множеств

Например, в случае двух множеств [math]~A, B[/math] формула включения-исключения имеет вид:

[math] | A \cup B | = | A | + | B | - | A \cap B |[/math]

В сумме [math]~| A | + | B |[/math] элементы пересечения [math]A \cap B[/math] учтены дважды, и чтобы компенсировать это мы вычитаем [math] | A \cap B |[/math] из правой части формулы. Справедливость этого рассуждения видна из диаграммы Эйлера-Венна для двух множеств, приведенной на рисунке справа.

Таким же образом и в случае [math]~n\gt 2[/math] множеств процесс нахождения количества элементов объединения [math]A_1 \cup A_2 \cup \ldots \cup A_n[/math] состоит во включении всего, затем исключении лишнего, затем включении ошибочно исключенного и так далее, то есть в попеременном включении и исключении. Отсюда и происходит название формулы.

Теорема

Пусть [math] A = \bigcup_{i=1}^{n}A_i [/math] , тогда по формуле включения-исключения:

[math] | A | = \sum_{I=(i_1,i_2, \ldots ,i_k) \subset \{ 1,2, \ldots ,n \} } (-1)^{k+1} \Big| \bigcap_{ j \in I } A_j \Big| [/math]

Доказательство

Для случая [math]~n=1[/math] и [math]~n=2[/math] теорема, очевидно, верна.

Теперь рассмотрим [math]~n\gt 2[/math]:

[math] A = \bigcup_{i=1}^{n}A_i = \Bigg( \underbrace {\bigcup_{i=1}^{n-1}A_i}_{B} \Bigg) \cup A_n [/math]


[math] | B | = \sum_{I \subset \{ 1,2, \ldots ,n-1 \} } (-1)^{|I|+1} \Big| \bigcap_{ j \in I } A_j \Big| [/math]


[math] | A | = | B | + | A_n | - | B \cap A_n |[/math]


[math] \Big| B \bigcap A_n \Big| = \Bigg| \Bigg( \bigcup_{i=1}^{n-1}A_i \Bigg) \bigcap A_n \Bigg|= \Bigg| \bigcup_{i=1}^{n-1} \bigg( A_i \bigcap A_n \bigg) \Bigg| = [/math]


[math] = \sum_{I \subset \{ 1,2, \ldots ,n-1 \} } (-1)^{|I|+1} \bigg| \bigcap_{ j \in I } \Big( A_j \bigcap A_n \Big) \bigg| = \sum_{I \subset \{ 1,2, \ldots ,n-1 \} } (-1)^{|I|+1} \Big| \bigcap_{ j\in I \cup \{ n \} } A_j \Big| [/math]


Таким образом:


[math] | A | = | A_n| + \Bigg( \sum_{I \subset \{ 1,2, \ldots ,n-1 \} } (-1)^{|I|+1} \Big| \bigcap_{ j \in I } A_j \Big| \Bigg) - \Bigg( \sum_{I \subset \{ 1,2, \ldots ,n-1 \} } (-1)^{|I|+1} \Big| \bigcap_{ j\in I \cup \{ n \} } A_j \Big| \Bigg) = \sum_{I \subset \{ 1,2, \ldots ,n \} } (-1)^{k+1} \Big| \bigcap_{ j \in I } A_j \Big| [/math]

Доказательство

Для случая [math]~n=1[/math] и [math]~n=2[/math] теорема, очевидно, верна.

Теперь рассмотрим [math]~n\gt 2[/math]:

[math] A = \bigcup_{i=1}^{n}A_i = \Bigg( \underbrace {\bigcup_{i=1}^{n-1}A_i}_{B} \Bigg) \cup A_n [/math]


[math] | B | = \sum_{I \subset \{ 1,2, \ldots ,n-1 \} } (-1)^{|I|+1} \Big| \bigcap_{ j \in I } A_j \Big| [/math]


[math] | A | = | B | + | A_n | - | B \cap A_n |[/math]


[math] \Big| B \bigcap A_n \Big| = \Bigg| \Bigg( \bigcup_{i=1}^{n-1}A_i \Bigg) \bigcap A_n \Bigg|= \Bigg| \bigcup_{i=1}^{n-1} \bigg( A_i \bigcap A_n \bigg) \Bigg| = [/math]


[math] = \sum_{I \subset \{ 1,2, \ldots ,n-1 \} } (-1)^{|I|+1} \bigg| \bigcap_{ j \in I } \Big( A_j \bigcap A_n \Big) \bigg| = \sum_{I \subset \{ 1,2, \ldots ,n-1 \} } (-1)^{|I|+1} \Big| \bigcap_{ j\in I \cup \{ n \} } A_j \Big| [/math]


Таким образом:


[math] | A | = | A_n| + \Bigg( \sum_{I \subset \{ 1,2, \ldots ,n-1 \} } (-1)^{|I|+1} \Big| \bigcap_{ j \in I } A_j \Big| \Bigg) - \Bigg( \sum_{I \subset \{ 1,2, \ldots ,n-1 \} } (-1)^{|I|+1} \Big| \bigcap_{ j\in I \cup \{ n \} } A_j \Big| \Bigg) = \sum_{I \subset \{ 1,2, \ldots ,n \} } (-1)^{k+1} \Big| \bigcap_{ j \in I } A_j \Big| [/math]

Теорема

Пусть [math] A = \bigcup_{i=1}^{n}A_i [/math] , тогда по формуле включения-исключения:

[math] | A | = \sum_{I=(i_1,i_2, \ldots ,i_k) \subset \{ 1,2, \ldots ,n \} } (-1)^{k+1} \Big| \bigcap_{ j \in I } A_j \Big| [/math]

Доказательство

Для случая [math]~n=1[/math] и [math]~n=2[/math] теорема, очевидно, верна.

Теперь рассмотрим [math]~n\gt 2[/math]:

[math] A = \bigcup_{i=1}^{n}A_i = \Bigg( \underbrace {\bigcup_{i=1}^{n-1}A_i}_{B} \Bigg) \cup A_n [/math]


[math] | B | = \sum_{I \subset \{ 1,2, \ldots ,n-1 \} } (-1)^{|I|+1} \Big| \bigcap_{ j \in I } A_j \Big| [/math]


[math] | A | = | B | + | A_n | - | B \cap A_n |[/math]


[math] \Big| B \bigcap A_n \Big| = \Bigg| \Bigg( \bigcup_{i=1}^{n-1}A_i \Bigg) \bigcap A_n \Bigg|= \Bigg| \bigcup_{i=1}^{n-1} \bigg( A_i \bigcap A_n \bigg) \Bigg| = [/math]


[math] = \sum_{I \subset \{ 1,2, \ldots ,n-1 \} } (-1)^{|I|+1} \bigg| \bigcap_{ j \in I } \Big( A_j \bigcap A_n \Big) \bigg| = \sum_{I \subset \{ 1,2, \ldots ,n-1 \} } (-1)^{|I|+1} \Big| \bigcap_{ j\in I \cup \{ n \} } A_j \Big| [/math]


Таким образом:


[math] | A | = | A_n| + \Bigg( \sum_{I \subset \{ 1,2, \ldots ,n-1 \} } (-1)^{|I|+1} \Big| \bigcap_{ j \in I } A_j \Big| \Bigg) + \Bigg( \sum_{I \subset \{ 1,2, \ldots ,n-1 \} } (-1)^{|I|+1} \Big| \bigcap_{ j\in I \cup \{ n \} } A_j \Big| \Bigg) = \sum_{I \subset \{ 1,2, \ldots ,n \} } (-1)^{k+1} \Big| \bigcap_{ j \in I } A_j \Big| [/math]